Last visit was: 21 May 2024, 15:47 It is currently 21 May 2024, 15:47
Close
GMAT Club Daily Prep
Thank you for using the timer - this advanced tool can estimate your performance and suggest more practice questions. We have subscribed you to Daily Prep Questions via email.

Customized
for You

we will pick new questions that match your level based on your Timer History

Track
Your Progress

every week, we’ll send you an estimated GMAT score based on your performance

Practice
Pays

we will pick new questions that match your level based on your Timer History
Not interested in getting valuable practice questions and articles delivered to your email? No problem, unsubscribe here.
Close
Request Expert Reply
Confirm Cancel
SORT BY:
Date
Tags:
Difficulty: 555-605 Level,    Assumption,                
Show Tags
Hide Tags
Senior Manager
Senior Manager
Joined: 24 Dec 2022
Posts: 293
Own Kudos [?]: 149 [0]
Given Kudos: 222
Send PM
Tutor
Joined: 16 Oct 2010
Posts: 14891
Own Kudos [?]: 65407 [0]
Given Kudos: 431
Location: Pune, India
Send PM
Tutor
Joined: 16 Oct 2010
Posts: 14891
Own Kudos [?]: 65407 [1]
Given Kudos: 431
Location: Pune, India
Send PM
VP
VP
Joined: 15 Dec 2016
Posts: 1365
Own Kudos [?]: 213 [0]
Given Kudos: 189
Send PM
Editorial: Our city's public transportation agency is facing a budget [#permalink]
avigutman wrote:
jabhatta2 wrote:

Quote:
Editorial: Our city's public transportation agency is facing a budget shortfall. The fastest growing part of the budget has been employee retirement benefits, which are exceptionally generous. Unless the budget shortfall is resolved, transportation service will be cut, and many transportation employees will lose their jobs. Thus, it would be in the employees' best interest for their union to accept cuts in retirement benefits.

Which of the following is an assumption the editorial's argument requires?

(A) The union should not accept cuts in retirement benefits if doing so would not be in the employees' best interest.


2nd attempt) Here is my attempt on why (A) is wrong

I first have to re-phrase (A) as the following

Quote:
Re-phrased (A) : the Union should accept (cuts in retirement benefits) ONLY IF (cuts in retirement benefits) ARE IN THE INTEREST OF employees.


I dont think we need re-phrased (A) to be 100 % true to go from the red to the blue. I am not able to enunciate why though

The blue font makes a claim about what would be in the employees' best interest, jabhatta2. Not about what the union's interests are, nor about what the union should do. For example, it's possible that the union doesn't care at all about the employees' best interest, and that wouldn't undermine the editorial's conclusion.


Thanks so much avigutman. Interesting point above in the yellow.

I think what you are saying is
  • Between the blue and the red : it doesn’t really matter what the union thinks or does | should do or should not do
  • The unions thoughts / interests are MOOT in order to go from the blue to the red
  • If (A – variant) was there, even (A- variant) would also be wrong

Quote:
A – variant: The union should not accept cuts in retirement benefits if doing so would not be in the employees' best interest.


(A-variant) if anything is perhaps a strengthener

Originally posted by jabhatta2 on 06 Jan 2023, 06:53.
Last edited by jabhatta2 on 06 Jan 2023, 19:31, edited 1 time in total.
Tutor
Joined: 17 Jul 2019
Posts: 1301
Own Kudos [?]: 2226 [0]
Given Kudos: 66
Location: Canada
GMAT 1: 780 Q51 V45
GMAT 2: 780 Q50 V47
GMAT 3: 770 Q50 V45
Send PM
Re: Editorial: Our city's public transportation agency is facing a budget [#permalink]
Expert Reply
jabhatta2 wrote:
Thanks so much avigutman. Interesting point above in the yellow.

I think what you are saying is
  • Between the blue and the red : it doesn’t really matter what the union thinks or does | should do or should not do
  • The unions thoughts / interests are MOOT in order to go from the blue to the red
  • If (A – variant) was there, even (A- variant) would also be wrong

Quote:
A – variant: The union should not accept cuts in retirement benefits if doing so would not be in the employees' best interest.


(A-variant) if anything is perhaps a strengthener

Can you try to articulate why or how (A-variant) strengthens the argument, jabhatta2?

Posted from my mobile device
VP
VP
Joined: 15 Dec 2016
Posts: 1365
Own Kudos [?]: 213 [0]
Given Kudos: 189
Send PM
Editorial: Our city's public transportation agency is facing a budget [#permalink]
avigutman wrote:
jabhatta2 wrote:

Quote:
Editorial: Our city's public transportation agency is facing a budget shortfall. The fastest growing part of the budget has been employee retirement benefits, which are exceptionally generous. Unless the budget shortfall is resolved, transportation service will be cut, and many transportation employees will lose their jobs. Thus, it would be in the employees' best interest for their union to accept cuts in retirement benefits.

Which of the following is an assumption the editorial's argument requires?

(A) The union should not accept cuts in retirement benefits if doing so would not be in the employees' best interest.


Thanks so much avigutman. Interesting point above in the yellow.

I think what you are saying is
  • Between the blue and the red : it doesn’t really matter what the union thinks or does | should do or should not do
  • The unions thoughts / interests are MOOT in order to go from the blue to the red
  • If (A – variant) was there, even (A- variant) would also be wrong

Quote:
A – variant: The union should not accept cuts in retirement benefits if doing so would not be in the employees' best interest.


(A-variant) if anything is perhaps a strengthener

Can you try to articulate why or how (A-variant) strengthens the argument, jabhatta2?

Posted from my mobile device


Hi avigutman – to go from the red to the blue, I think you need the following necessary assumptions
(i) The author of the argument believes the union has the employee’s best interest (hence the recommendation in the blue)
(ii) Saving jobs is in the best interest for employees

I got the sense that (A-variant) strengthened Necessary Assumption (1)
Tutor
Joined: 17 Jul 2019
Posts: 1301
Own Kudos [?]: 2226 [0]
Given Kudos: 66
Location: Canada
GMAT 1: 780 Q51 V45
GMAT 2: 780 Q50 V47
GMAT 3: 770 Q50 V45
Send PM
Re: Editorial: Our city's public transportation agency is facing a budget [#permalink]
Expert Reply
jabhatta2 wrote:
Hi avigutman – to go from the red to the blue, I think you need the following necessary assumptions
(i) The author of the argument believes the union has the employee’s best interest (hence the recommendation in the blue)
(ii) Saving jobs is in the best interest for employees

I got the sense that (A-variant) strengthened Necessary Assumption (1)

It would be in my best interest for you to give me a million dollars, jabhatta2. That’s true whether or not I believe that you care about my best interest. (i) isn’t necessary.

Posted from my mobile device
VP
VP
Joined: 15 Dec 2016
Posts: 1365
Own Kudos [?]: 213 [0]
Given Kudos: 189
Send PM
Editorial: Our city's public transportation agency is facing a budget [#permalink]
Hi avigutman - the analogy did not help unfortunately. :(

Question : You mention (I) is not a necessary assumption.

What necessary assumptions do you think the editor is making with regards to the union specifically ?

I would imagine there would be at-least one assumption regarding the union specifically.

I would imagine the introduction of a third entity (not the editor / not the employees) but the union would bring about at least one assumption the editors have made regarding the union specifically

If there are no assumptions w.r.t the union specifically - i dont understand what the phrase "for their union" is doing in the blue conclusion (Yellow highlight below).

Quote:
Editorial: Our city's public transportation agency is facing a budget shortfall. The fastest growing part of the budget has been employee retirement benefits, which are exceptionally generous. Unless the budget shortfall is resolved, transportation service will be cut, and many transportation employees will lose their jobs. Thus, it would be in the employees' best interest for their union to accept cuts in retirement benefits.

Which of the following is an assumption the editorial's argument requires?

(A) The union should not accept cuts in retirement benefits if doing so would not be in the employees' best interest.
Tutor
Joined: 17 Jul 2019
Posts: 1301
Own Kudos [?]: 2226 [0]
Given Kudos: 66
Location: Canada
GMAT 1: 780 Q51 V45
GMAT 2: 780 Q50 V47
GMAT 3: 770 Q50 V45
Send PM
Re: Editorial: Our city's public transportation agency is facing a budget [#permalink]
Expert Reply
jabhatta2 wrote:
What necessary assumptions do you think the editor is making with regards to the union specifically ?

I would imagine there would be at-least one assumption regarding the union specifically.

I would imagine the introduction of a third entity (not the editor / not the employees) but the union would bring about at least one assumption the editors have made regarding the union specifically
Why, jabhatta2?
jabhatta2 wrote:
If there are no assumptions w.r.t the union specifically - i dont understand what the phrase "for their union" is doing in the blue conclusion (Yellow highlight below).

Which of the following is the editorial's claim, jabhatta2, in your opinion? You can choose none, one, or more than one of the following:
(a) The union should accept cuts in retirement benefits.
(b) The union's job is to do what's in the employees' best interest.
(c) If the union does accept cuts in retirement benefits, that would be in the employees' best interest.
VP
VP
Joined: 15 Dec 2016
Posts: 1365
Own Kudos [?]: 213 [0]
Given Kudos: 189
Send PM
Editorial: Our city's public transportation agency is facing a budget [#permalink]
avigutman wrote:
jabhatta2 wrote:
What necessary assumptions do you think the editor is making with regards to the union specifically ?

I would imagine there would be at-least one assumption regarding the union specifically.

I would imagine the introduction of a third entity (not the editor / not the employees) but the union would bring about at least one assumption the editors have made regarding the union specifically
Why, jabhatta2?
jabhatta2 wrote:
If there are no assumptions w.r.t the union specifically - i dont understand what the phrase "for their union" is doing in the blue conclusion (Yellow highlight below).

Which of the following is the editorial's claim, jabhatta2, in your opinion? You can choose none, one, or more than one of the following:
(a) The union should accept cuts in retirement benefits.
(b) The union's job is to do what's in the employees' best interest.
(c) If the union does accept cuts in retirement benefits, that would be in the employees' best interest.


Hi avigutman

(I) Yes, (a) is a claim by the editor

(II) Subtle difference. (b) is not a claim by the editor. However, I think (b) is in itself an hopeful assumption made by the editor when he makes the blue conclusion.

(III) Yes, (c) is a claim by the editor
Tutor
Joined: 17 Jul 2019
Posts: 1301
Own Kudos [?]: 2226 [0]
Given Kudos: 66
Location: Canada
GMAT 1: 780 Q51 V45
GMAT 2: 780 Q50 V47
GMAT 3: 770 Q50 V45
Send PM
Re: Editorial: Our city's public transportation agency is facing a budget [#permalink]
Expert Reply
jabhatta2 wrote:
avigutman wrote:
jabhatta2 wrote:
What necessary assumptions do you think the editor is making with regards to the union specifically ?

I would imagine there would be at-least one assumption regarding the union specifically.

I would imagine the introduction of a third entity (not the editor / not the employees) but the union would bring about at least one assumption the editors have made regarding the union specifically
Why, jabhatta2?
jabhatta2 wrote:
If there are no assumptions w.r.t the union specifically - i dont understand what the phrase "for their union" is doing in the blue conclusion (Yellow highlight below).

Which of the following is the editorial's claim, jabhatta2, in your opinion? You can choose none, one, or more than one of the following:
(a) The union should accept cuts in retirement benefits.
(b) The union's job is to do what's in the employees' best interest.
(c) If the union does accept cuts in retirement benefits, that would be in the employees' best interest.


Hi avigutman

(I) Yes, (a) is a claim by the editor

(II) Subtle difference. (b) is not a claim by the editor. However, I think (b) is in itself an hopeful assumption made by the editor when he makes the blue conclusion.

(III) Yes, (c) is a claim by the editor

Imagine that someone doesn’t see (a) being claimed by the editorial. What would you point to, jabhatta2, to convince that person. How would you prove that the editorial is claiming (a)?

Posted from my mobile device
VP
VP
Joined: 15 Dec 2016
Posts: 1365
Own Kudos [?]: 213 [0]
Given Kudos: 189
Send PM
Editorial: Our city's public transportation agency is facing a budget [#permalink]
avigutman wrote:
jabhatta2 wrote:
avigutman wrote:
Which of the following is the editorial's claim, jabhatta2, in your opinion? You can choose none, one, or more than one of the following:
(a) The union should accept cuts in retirement benefits.
(b) The union's job is to do what's in the employees' best interest.
(c) If the union does accept cuts in retirement benefits, that would be in the employees' best interest.


Hi avigutman

(I) Yes, (a) is a claim by the editor

(II) Subtle difference. (b) is not a claim by the editor. However, I think (b) is in itself an hopeful assumption made by the editor when he makes the blue conclusion.

(III) Yes, (c) is a claim by the editor

Imagine that someone doesn’t see (a) being claimed by the editorial. What would you point to, jabhatta2, to convince that person. How would you prove that the editorial is claiming (a)?

Posted from my mobile device


avigutman -
I would point to the phrase "for their union" (yellow highlight) in the blue conclusion.

Why include that phrase (For their union) in the conclusion, if the editor does not want or hope or expect that the union eventually proceed with the cuts in retirement benefits

-----------------

Conclusion -- Thus, it would be in the employees' best interest for their union to accept cuts in retirement benefits.
VP
VP
Joined: 15 Dec 2016
Posts: 1365
Own Kudos [?]: 213 [0]
Given Kudos: 189
Send PM
Editorial: Our city's public transportation agency is facing a budget [#permalink]
GMATNinja wrote:
Here's (A):
Quote:
(A) The transportation employees' union should not accept cuts in retirement benefits if doing so would not be in the employees' best interest.

This doesn't absolutely HAVE to be true in order for the author to reach his/her conclusion. Perhaps there IS a circumstance under which it makes sense for the union to accept retirement cuts, even if it's not in the employees' best interests.

Maybe, for example, the cut in retirement benefits would directly fund some other really important project (building a new hospital or something). In this case, it's possible that the transportation union SHOULD support the cut in benefits even though it ISN'T in the best interests of the employees.

That doesn't impact the author's conclusion that it WOULD be in the best interest of the employees to accept THESE particular cuts in retirement benefits.

Because the argument doesn't depend on (A), (A) is not the correct answer.



Hi GMATNinja - would you agree that (A) would be 'helpful" if it true ?

I think from what I read above - (A) is a strengthener but not a necessary assumption

If you could respond ASAP - that would be great ! D - Day is coming up :(
VP
VP
Joined: 15 Dec 2016
Posts: 1365
Own Kudos [?]: 213 [0]
Given Kudos: 189
Send PM
Editorial: Our city's public transportation agency is facing a budget [#permalink]
Analogy :

Editorial – Avi was caught stealing at work. The company is looking to fire Avi. If Avi is fired, Avi will never work again. The company has however, offered Avi a deal ,in which, Avi pays a 5,000 dollar fine but no firing. Thus, it would be in Avi’s best interest for his union to accept the deal.

Which of the following is an assumption the editorial's argument requires?

(Option A) Avi’s union should not accept the deal if doing so would not be in the best interest of Avi.
(A-variant) Avi’s union should accept the deal if doing so is in the best interest of Avi.
Tutor
Joined: 17 Jul 2019
Posts: 1301
Own Kudos [?]: 2226 [0]
Given Kudos: 66
Location: Canada
GMAT 1: 780 Q51 V45
GMAT 2: 780 Q50 V47
GMAT 3: 770 Q50 V45
Send PM
Re: Editorial: Our city's public transportation agency is facing a budget [#permalink]
Expert Reply
jabhatta2 wrote:
Why include that phrase (For their union) in the conclusion, if the editor does not want or hope or expect that the union eventually proceed with the cuts in retirement benefits

The union is mentioned in the conclusion because it has the authority to accept or reject the cuts in the retirement benefits.
The argument isn't claiming that the union will or won't or should or shouldn't do anything, jabhatta2.
The argument is merely claiming that if the union was to accept the cuts, such a move would be in the employees' best interest.
Does the author claim that the union should accept the cuts? No. Does the author claim that the union will accept the cuts? No.
Therefore, the argument doesn't depend on any assumptions about the union. The claim isn't about the union. Rather, it's about what would or wouldn't be in the employees' best interest.
Intern
Intern
Joined: 15 Oct 2022
Posts: 1
Own Kudos [?]: 0 [0]
Given Kudos: 24
Send PM
Re: Editorial: Our city's public transportation agency is facing a budget [#permalink]
aggvipul wrote:
TheRzS wrote:
Could an expert please weigh in?

Q1.
A. The transportation employees' union should not accept cuts in retirement benefits if doing so would not be in the employees' best interest.
E. The transportation employees' union will not accept cuts in retirement benefits if doing so will not allow more transportation employees to keep their jobs.

In A and E, does "doing so" stand for ACCEPTING or NOT ACCEPTING?

Q2.
Could someone please try negating A and E? Negating them seems to hurt the argument.

Thanks and regards

TheRzS Q1 -"doing so" here is indicating "Accepting"
Q2- Negating the option A. rather warps the information or the intent of what is being said, try this
A1. The transportation employees' union should accept cuts in retirement benefits if doing so would not be in the employees' best interest- it rather now intents to say that employees should accept loss
A2. The transportation employees' union should not accept cuts in retirement benefits if doing so would be in the employees' best interest- same here, it intents to say that employees should accept loss
Therefore, it does not hurt the argument rather warps the gist of question stem
would be happy to help in case anything is still unclear




Hi

My 2 cents

I believe Option A only mention only about employee benefits. and option E mentions whole about solving the issue of budget deficit.
VP
VP
Joined: 15 Dec 2016
Posts: 1365
Own Kudos [?]: 213 [0]
Given Kudos: 189
Send PM
Editorial: Our city's public transportation agency is facing a budget [#permalink]
Hi ReedArnoldMPREP ChiranjeevSingh AnishPassi - how would you go about knocking out (A) and (E)

have read a lot of the above comments but still not 100 % confident about (A) and (E)

Do you use LSAT theory to re-phrase (a) and (e), first perhaps ?

When I read (A) and (E) - i immediately infer/rephrase (A) & (E) as stating :

Unions have the back of their employees

I keep thinking of the blue statement whenever i read (A) and (E)

Given the blue is in my mind, when I read option (A) and (E) -- i keep getting drawn to option (A) and option (E)

Reason - My statement in blue has to be something that is true in order for the conclusion to be true.
VP
VP
Joined: 15 Dec 2016
Posts: 1365
Own Kudos [?]: 213 [0]
Given Kudos: 189
Send PM
Editorial: Our city's public transportation agency is facing a budget [#permalink]
avigutman wrote:
jabhatta2 wrote:
Why include that phrase (For their union) in the conclusion, if the editor does not want or hope or expect that the union eventually proceed with the cuts in retirement benefits

The union is mentioned in the conclusion because it has the authority to accept or reject the cuts in the retirement benefits.
The argument isn't claiming that the union will or won't or should or shouldn't do anything, jabhatta2.
The argument is merely claiming that if the union was to accept the cuts, such a move would be in the employees' best interest.
Does the author claim that the union should accept the cuts? No. Does the author claim that the union will accept the cuts? No.
Therefore, the argument doesn't depend on any assumptions about the union. The claim isn't about the union. Rather, it's about what would or wouldn't be in the employees' best interest.


Hi avigutman – I think I am struggling to swallow the bits in yellow.

I keep bringing in the intentions of the union ITSELF for some reason or the other (I don’t know).

Remember in CR - every phrase -- is critical and every phrase is placed for a reason

In fact - the moment I remove the phrase “for their union” from the argument (Screenshot below) – I THEN BELIEVE WHAT you say in the yellow bits above.

Then only I can see why (a) and (e) are irrelevant

But the phrase is there and is confusing :(
Attachments

screenshot 2.jpg
screenshot 2.jpg [ 109.56 KiB | Viewed 585 times ]

Tutor
Joined: 17 Jul 2019
Posts: 1301
Own Kudos [?]: 2226 [0]
Given Kudos: 66
Location: Canada
GMAT 1: 780 Q51 V45
GMAT 2: 780 Q50 V47
GMAT 3: 770 Q50 V45
Send PM
Re: Editorial: Our city's public transportation agency is facing a budget [#permalink]
Expert Reply
jabhatta2 wrote:
I keep bringing in the intentions of the union ITSELF for some reason or the other (I don’t know).

Remember in CR - every phrase -- is critical and every phrase is placed for a reason

In fact - the moment I remove the phrase “for their union” from the argument (Screenshot below) – I THEN BELIEVE WHAT you say in the yellow bits above.

Then only I can see why (a) and (e) are irrelevant

But the phrase is there and is confusing :(

How would you rephrase the conclusion, in your own words, jabhatta2?
VP
VP
Joined: 15 Dec 2016
Posts: 1365
Own Kudos [?]: 213 [0]
Given Kudos: 189
Send PM
Editorial: Our city's public transportation agency is facing a budget [#permalink]
avigutman wrote:
How would you rephrase the conclusion, in your own words, jabhatta2?


avigutman hmm - i am almost tempted to just drop the phrase "for their union to accept" completely and making the conclusion MUCH EASIER

Re-phrased conclusion - Thus, employees should accept the cuts in retirement benefits.

If the blue was instead the conclusion -- what the union itself does / what the union itself thinks / what the unions own interests are -- become moot.

But dropping phrase to 'oversimply' an argument is a dangerous strategy as it has led to problems in OTHER CR questions.
GMAT Club Bot
Editorial: Our city's public transportation agency is facing a budget [#permalink]
   1   2   3   4   
Moderators:
GMAT Club Verbal Expert
6936 posts
GMAT Club Verbal Expert
238 posts
CR Forum Moderator
832 posts